Eufisky - The lost book

幸子解答的李炯生习题

原先我们自己给李炯生习题作出的解答略显繁琐,这里贴出幸子的一些解答。


设$f(x)$是$2n+1$次多项式, $n$为正整数, $f(x)+1$被${(x-1)}^n$整除,而$f(x)-1$被${(x+1)}^n$整除,求$f(x)$.


解:由题意,存在$2u\left( x \right),2v\left( x \right) \in \mathbb{F}\left[ x \right]$,使得\[f\left( x \right) = 2u\left( x \right){\left( {x + 1} \right)^n} + 1 = 2v\left( x \right){\left( {x - 1} \right)^n} - 1.\]即得\[v\left( x \right){\left( {x - 1} \right)^n} + w\left( x \right){\left( {x + 1} \right)^n} = 1,\tag{$\ast$}\]其中$\deg v\left( x \right) = \deg w\left( x \right) = n + 1,w\left( x \right) = - u\left( x \right)$.

若$(\ast)$存在特解$v_1(x)$和$w_1(x)$,则${\left( {x + 1} \right)^n}|v\left( x \right) - {v_1}\left( x \right)$且${\left( {x - 1} \right)^n}|w\left( x \right) - {w_1}\left( x \right)$.

 

存在多项式$\alpha(x)$使得

\[\left\{ {\begin{array}{*{20}{c}}{v\left( x \right) = {v_1}\left( x \right) + {{\left( {x + 1} \right)}^n}\alpha \left( x \right)}\\{w\left( x \right) = {w_1}\left( x \right) - {{\left( {x - 1} \right)}^n}\alpha \left( x \right)}\end{array}} \right.,\]

而对于任意的$\alpha(x)\in \mathbb{F}[x]$,由

$$\left\{ {\begin{array}{*{20}{c}}{v\left( x \right) = {v_0}\left( x \right) + {{\left( {x + 1} \right)}^n}\alpha \left( x \right)}\\{w\left( x \right) = {w_0}\left( x \right) - {{\left( {x - 1} \right)}^n}\alpha \left( x \right)}\end{array}} \right.$$

定义的$v(x),w(x)$均满足$(\ast)$.由上式定义的$v(x)$和$w(x)$为$(\ast)$通解,于是$v\left( x \right) = \left[ {1 - w\left( x \right){{\left( {x + 1} \right)}^n}} \right]{\left( {x - 1} \right)^{ - n}},w\left( x \right) = \left[ {1 - v\left( x \right){{\left( {x - 1} \right)}^n}} \right]{\left( {x + 1} \right)^{ - n}},v\left( { - 1} \right) = {\left( { - 2} \right)^{ - n}},w\left( 1 \right) = {2^{ - n}}$,

 

当$1\leq i\leq n-1$时,利用Leibniz公式

\begin{align*}{v^{\left( i \right)}}\left( { - 1} \right) &= \sum\limits_{k = 0}^i {{{\left[ {1 - w\left( x \right){{\left( {x + 1} \right)}^n}} \right]}^{\left( k \right)}}{{\left[ {{{\left( {x - 1} \right)}^{ - n}}} \right]}^{\left( {i - k} \right)}}} {|_{x = - 1}} = {\left( { - 1} \right)^n}{2^{ - n - i}}n \cdots \left( {n + i - 1} \right)\\{w^{\left( i \right)}}\left( 1 \right) &= \sum\limits_{k = 0}^i {{{\left[ {1 - v\left( x \right){{\left( {x - 1} \right)}^n}} \right]}^{\left( k \right)}}{{\left[ {{{\left( {x + 1} \right)}^{ - n}}} \right]}^{\left( {i - k} \right)}}} {|_{x = 1}} = {\left( { - 1} \right)^i}{2^{ - n - i}}n \cdots \left( {n + i - 1} \right).\end{align*}

由Taylor公式,存在唯一的次数小于$n$的多项式\[{v_0}\left( x \right) = \frac{1}{{{{\left( { - 2} \right)}^n}}}\sum\limits_{i = 0}^{n - 1} {\frac{1}{{{2^i}}}C_{n + i - 1}^i{{\left( {x + 1} \right)}^i}} ,{w_0}\left( x \right) = \frac{1}{{{2^n}}}\sum\limits_{i = 0}^{n - 1} {\frac{1}{{{{\left( { - 2} \right)}^i}}}C_{n + i - 1}^i{{\left( {x - 1} \right)}^i}} \]使得$(\ast)$成立.

 

因此对任意的一次多项式$\alpha \left( x \right)$,

 

$f\left( x \right) = 2\left[ {{{\left( {x - 1} \right)}^n}\alpha \left( x \right) - {w_0}\left( x \right)} \right]{\left( {x + 1} \right)^n} + 1= 2\left[ {{v_0}\left( x \right) + {{\left( {x + 1} \right)}^n}\alpha \left( x \right)} \right]{\left( {x - 1} \right)^n} - 1$均满足题设.


设$A\in \mathbb{R}^{2n\times 2n}$,且$A\left( {\begin{array}{*{20}{c}}0&{{I_{\left( n \right)}}}\\{ - {I_{\left( n \right)}}}&0\end{array}} \right){A^T} = \left( {\begin{array}{*{20}{c}}0&{{I_{\left( n \right)}}}\\{ - {I_{\left( n \right)}}}&0\end{array}} \right)$.证明: $\det A=1$.


证:令$U = \frac{1}{{\sqrt 2 }}\left( {\begin{array}{*{20}{c}}{{I_n}}&{ - i{I_n}}\\{{I_n}}&{i{I_n}}\end{array}} \right),A = \left( {\begin{array}{*{20}{c}}B&C\\D&G\end{array}} \right)$,

\[T = UA{U^H} = \frac{1}{2}\left( {\begin{array}{*{20}{c}}{B + G + \left( {C - D} \right)i}&{B - G - \left( {C + D} \right)i}\\{B - G + \left( {C + D} \right)i}&{B + G - \left( {C - D} \right)i}\end{array}} \right) = \left( {\begin{array}{*{20}{c}}P&Q\\{\bar Q}&{\bar P}\end{array}} \right).\]

由$A\left( {\begin{array}{*{20}{c}}O&{{I_n}}\\{ - {I_n}}&O\end{array}} \right){A^T} = \left( {\begin{array}{*{20}{c}}O&{{I_n}}\\{{I_n}}&O\end{array}} \right)$知

\[\left( {\begin{array}{*{20}{c}}P&Q\\{\bar Q}&{\bar P}\end{array}} \right)\left( {\begin{array}{*{20}{c}}{{I_n}}&O\\O&{ - {I_n}}\end{array}} \right)\left( {\begin{array}{*{20}{c}}{{P^H}}&{{Q^T}}\\{{Q^H}}&{{P^T}}\end{array}} \right) = \left( {\begin{array}{*{20}{c}}{{I_n}}&O\\O&{ - {I_n}}\end{array}} \right).\]

即$P{P^H} - Q{Q^H} = {I_n}$及$P{Q^T} = Q{P^T}$,注意到$P{P^H} = {I_n} + Q{Q^H} > 0$,即$\det P\neq0$,且$\det A=\det T$.而$\det \left( {\begin{array}{*{20}{c}}P&Q\\{\bar Q}&{\bar P}\end{array}} \right)\left( {\begin{array}{*{20}{c}}{{P^H}}&O\\{ - {Q^H}}&{{I_n}}\end{array}} \right) = \det \left( {\begin{array}{*{20}{c}}{{I_n}}&Q\\O&{\bar P}\end{array}} \right)$.因此$\det A=1$.


 

曲线积分的计算

张元博问了这么一道题:计算曲线积分\[\oint_C {\left( {{{\left( {x + 1} \right)}^2} + {{\left( {y - 2} \right)}^2}} \right)ds} ,\]其中$C$表示曲面$x^2+y^2+z^2=1$与$x+y+z=1$的交线.


解:我是利用常规的三角换元解决的.联立方程有

\[\left\{ \begin{array}{l}{x^2} + {y^2} + {z^2} = 1\\x + y + z = 1\end{array} \right. \Rightarrow {\left( {x - \frac{1}{2} + \frac{y}{2}} \right)^2} + \frac{3}{4}{\left( {y - \frac{1}{3}} \right)^2} = \frac{1}{3}.\]
\[\left\{ \begin{array}{l}x - \frac{1}{2} + \frac{y}{2} = \frac{1}{{\sqrt 3 }}\cos \theta \\\frac{{\sqrt 3 }}{2}\left( {y - \frac{1}{3}} \right) = \frac{1}{{\sqrt 3 }}\sin \theta \end{array} \right. \Rightarrow \left\{ \begin{array}{l}x = \frac{1}{3} - \frac{1}{3}\sin \theta  + \frac{1}{{\sqrt 3 }}\cos \theta \\y = \frac{1}{3} + \frac{2}{3}\sin \theta \\z = 1 - x - y = \frac{1}{3} - \frac{1}{3}\sin \theta  - \frac{1}{{\sqrt 3 }}\cos \theta \end{array} \right.\]
则\[ds = \sqrt {{{\left( {x'\left( \theta  \right)} \right)}^2} + {{\left( {y'\left( \theta  \right)} \right)}^2} + {{\left( {z'\left( \theta  \right)} \right)}^2}}  = \frac{{\sqrt 6 }}{3}.\]
因此
\begin{align*}&\oint_C {\left( {{{\left( {x + 1} \right)}^2} + {{\left( {y - 2} \right)}^2}} \right)ds} \\= &\frac{{\sqrt 6 }}{3}\int_0^{2\pi } {\left( {\frac{5}{9}{{\sin }^2}\theta  + \frac{1}{3}{{\cos }^2}\theta  - \frac{{28}}{9}\sin \theta  + \frac{8}{{3\sqrt 3 }}\cos \theta  - \frac{2}{{3\sqrt 3 }}\sin \theta \cos \theta  + \frac{{41}}{9}} \right)d\theta } \\= &\frac{{\sqrt 6 }}{3} \times 5 = \frac{{5\sqrt 6 }}{3}.\end{align*}

 

余神题解

二重积分难题荟萃

第一个是09年西北大学的考研题,也在史济怀老师的数分书上找得到.这里的解答就是史老爷子自己的.


计算积分\[\iint_D {\frac{1}{{xy\left( {\ln^2 x + {{\ln }^2}y} \right)}}} dxdy,D = \left\{ {\left( {x,y} \right)\left| {x + y \ge 1,{x^2} + {y^2} \le 1} \right.} \right\}.\]


enlightened解.作变换$x=e^{r\cos\theta},y=e^{r\sin\theta}$,则

\[\frac{{\partial \left( {x,y} \right)}}{{\partial \left( {r,\theta } \right)}} = \left| {\begin{array}{*{20}{c}}{\cos \theta {e^{r\cos \theta }}}&{ - r\sin \theta {e^{r\cos \theta }}}\\{\sin \theta {e^{r\sin \theta }}}&{r\cos \theta {e^{r\sin \theta }}}\end{array}} \right| = r{e^{r\sin \theta }}{e^{r\cos \theta }}.\]

原积分变为\[I = \iint_\Delta {\frac{{drd\theta }}{r}} .\]

这里的$\Delta$是变换以后的积分区域.注意$x+y=1$和$x^2+y^2=1$分别被变为

\[\left\{ \begin{array}{l}{e^{r\cos \theta }} + {e^{r\sin \theta }} = 1,\\{e^{2r\cos \theta }} + {e^{2r\sin \theta }} = 1.\end{array} \right.\]

 

现在来分析由上述两条曲线所围成的$(r,\theta)$平面上的区域是什么形状.从第一个式子可以看出$\theta$的变化范围必须使$\cos \theta$和$\sin \theta$都取负值,故$\theta$只能在$\left[\pi,\frac32\pi \right]$中取值.

 

假设由第一个式子确定的函数为$r=r(\theta)$,则由第二个式子确定的函数便为$r=\frac12 r(\theta)$.因此

 

\[I = \iint_\Delta {\frac{{drd\theta }}{r}} = \int_\pi ^{\frac{3}{2}\pi } {d\theta } \int_{\frac{1}{2}r\left( \theta \right)}^{r\left( \theta \right)} {\frac{{dr}}{r}} = \frac{\pi }{2}\ln 2.\]

 

苹果公司总部


 

史济怀复变习题解答

本文主要给出史济怀老师复变函数的某些习题解答,主要来自陶哲轩小弟。


然后对后面的函数用最大模原理.




 

2015年浙大数分题

1. 求 $\lim\limits_{n\rightarrow+\infty} \dfrac{(n^2+1)(n^2+2)..(n^2+n)}{(n^2-1)(n^2-2)..(n^2-n)}$.
 
2. 求$\displaystyle\lim\limits_{x \rightarrow 0+} \dfrac{1}{x^5}\int_0^x e^{-t^2}dt+\dfrac{1}{3}\dfrac{1}{x^2}-\dfrac{1}{x^4}$.
 
3. $\displaystyle I(r)=\oint \dfrac{y}{x^2+y^2}\mathrm{d}x-\frac{x}{x^2+y^2}\mathrm{d}y$, 其中曲线方程为$x^2+y^2+xy=r^2$, 取正方向, 求$\lim\limits_{r\rightarrow\infty}I(r)$.
 
4. 求$\displaystyle\int_{e^{-2n\pi}}^0 \sin\ln\dfrac{1}{x}\mathrm{d}x$.
 
5. 考察黎曼函数的连续性, 可微性, 黎曼可积性.
 
6. 在$\mathbb{R}^n$中, $f$为定义在某个区域上的一个函数, 有一阶连续偏导, 且偏导数有界. 
证明:
(1) 若$D$为凸区域证明$f$一致连续. 
(2)考察$D$不是凸区域的情况.
 
7. $\{f_n\}$为一个连续函数列, 且对于任意给定的$x$, $\{f_n(x)\}$有界, 证明存在一个小区间在此小区间内$f_n$一致有界.
 
8. (1) 证明$\Gamma(s)$在 $(0,\infty$内无穷次可微.
(2) 证明$\Gamma(s)$ , $\ln(\Gamma)$都是严格凸函数.
 
9. $f$ 二阶可微, 且$f$, $f'$, $f''$ 都大于等于$0$, 且存在一个正数$c$, $f''(x)\leq cf(x)$. 证明:
(1) $\displaystyle\lim\limits_{x\rightarrow-\infty}f(x)=0$;
(2) 证明存在正数$a$, 有$f'\leq af$,并求出$a$.
 
10. 证明 Fejer定理.
 
11. 设$f$在$[A, B]$上黎曼可积, $0<f<1$, 对于任意的$\varepsilon$, 构造一个函数$g$, 满足
(1) $g$是一个阶梯函数, 且取值只能为$0$或$1$.
(2) $\displaystyle\left| \int_a^b f-g \mathrm{d}x\right|<\varepsilon$, $a$, $b$ 属于$[A,B]$不等号关于$a$, $b$是一致的.

浙大14年数分题:Dirichlet引理的证明

Riemann-Lebesgue引理大家都很熟悉,那Dirichlet引理呢?


$f(x)$在$[0,1]$单增,证明:

\[\mathop {\lim }\limits_{y \to + \infty } \int_0^1 {f\left( x \right)\frac{{\sin xy}}{x}dx} = \frac{\pi }{2}f\left( {{0_ + }} \right).\]


这是Dirichlet引理,菲赫金哥尔茨的《微积分教程》第三卷P358有详细的证明.另外,汪林的《数学分析问题研究与评注》P147上有他的推广及其证明.


对任意给出的$\varepsilon>0$, $\exists 0<\delta<1$,使得对于$0<t\leq \delta$,

\[0 \le g\left( t \right) - g\left( {{0_ + }} \right) < M_1\varepsilon ,\]

其中$M_1$是任意给定的常数.

 

考察积分

\begin{align*}\int_0^1 {\left[ {f\left( x \right) - f\left( {{0_ + }} \right)} \right]\frac{{\sin xy}}{x}dx} &= \left( {\int_0^\delta {} + \int_\delta ^1 {} } \right)\left[ {f\left( x \right) - f\left( {{0_ + }} \right)} \right]\frac{{\sin xy}}{x}dx\\&= {I_1} + {I_2}.\end{align*}

 

对于$I_1$,运用积分第二中值定理,我们有

\[{I_1} = \left[ {f\left( \delta \right) - f\left( {{0_ + }} \right)} \right]\int_\eta ^\delta {\frac{{\sin xy}}{x}dx} = \left[ {f\left( \delta \right) - f\left( {{0_ + }} \right)} \right]\int_{y\eta }^{y\delta } {\frac{{\sin z}}{z}dz} ,\]

 

其中第二个因子对于一切值$y$一致有界.事实上,由反常积分$\displaystyle \int_0^\infty {\frac{{\sin z}}{z}dz}$的收敛性,可见当$z\to \infty$时, $z(z\geq 0)$的连续函数$\displaystyle \int_0^z {\frac{{\sin z}}{z}dz} $有有限的极限,并且对于一切值$z$有界

\[\left| {\int_0^z {\frac{{\sin z}}{z}dz} } \right| \le L\left( L \text{为常数}\right),\]从而

\[\left| {\int_{y\eta }^{y\delta } {\frac{{\sin z}}{z}dz} } \right| = \left| {\int_0^{y\delta } {} + \int_0^{y\eta } {} } \right| \le 2L.\]

 

对于第一个因子,取$M_1=\frac{1 }{{4L}}$,则有$f\left( \delta \right) - f\left( {{0_ + }} \right) < \frac{\varepsilon }{{4L}}$.

 

 

因此\[\left| {{I_1}} \right| \le \left[ {f\left( \delta \right) - f\left( {{0_ + }} \right)} \right]\left| {\int_{y\eta }^{y\delta } {\frac{{\sin z}}{z}dz} } \right| < \frac{\varepsilon }{{4L}} \cdot 2L = \frac{\varepsilon }{2}.\]

 

至于$I_2$,由于$\displaystyle \int_\delta ^1 {\frac{{f\left( x \right) - f\left( {{0_ + }} \right)}}{x}dx} $存在,由Riemann-Lebesgue引理可知$\mathop {\lim }\limits_{y \to \infty } {I_2} = 0$,即对$\varepsilon >0,\exists M_2>0$,使得$y>M_2$时,有$\left| {{I_2}} \right| < \frac{\varepsilon }{2}$.

 

因此\[\left| {\int_0^1 {\left[ {f\left( x \right) - f\left( {{0_ + }} \right)} \right]\frac{{\sin xy}}{x}dx} } \right| \le \left| {{I_1}} \right| + \left| {{I_2}} \right| < \varepsilon .\]

即\[\mathop {\lim }\limits_{y \to + \infty } \int_0^1 {\left[ {f\left( x \right) - f\left( {{0_ + }} \right)} \right]\frac{{\sin xy}}{x}dx} = 0.\]

 

从而

\begin{align*}&\mathop {\lim }\limits_{y \to + \infty } \int_0^1 {f\left( x \right)\frac{{\sin xy}}{x}dx} = \frac{\pi }{2}f\left( {{0_ + }} \right)\\=& \mathop {\lim }\limits_{y \to + \infty } \int_0^1 {\left[ {f\left( x \right) - f\left( {{0_ + }} \right)} \right]\frac{{\sin xy}}{x}dx} + f\left( {{0_ + }} \right)\mathop {\lim }\limits_{y \to + \infty } \int_0^1 {\frac{{\sin xy}}{x}dx} \\= &0 + f\left( {{0_ + }} \right)\int_0^{ + \infty } {\frac{{\sin z}}{z}dz} = \frac{\pi }{2}f\left( {{0_ + }} \right).\end{align*}

一个具有概率背景的积分题

计算\[\mathop {\lim }\limits_{n \to  + \infty } \frac{{1 + n + \frac{{{n^2}}}{{2!}} +  \cdots  + \frac{{{n^n}}}{{n!}}}}{{{e^n}}} = \frac{1}{2}.\]


解.

法二.(小米)利用概率, 就是$n$个独立的参数为1的指数分布(泊松分布)和小于等于$n$的概率根据中心极限定理概率收敛到$1/2$.

2015年武汉大学高等代数考研试题

源自:http://www.math.org.cn/forum.php?mod=viewthread&tid=31895&highlight=%E6%AD%A6%E6%B1%89%E5%A4%A7%E5%AD%A6

级数中的一些反例

判断题:若级数$\displaystyle \sum_{n=1}^{+\infty}a_n^3$ 收敛, 则$\displaystyle \sum_{n=1}^{+\infty}\frac{a_n}{n}$亦收敛.


wink反例:(老骥伏枥)令$a_{n}=\frac{1}{\log n}\sqrt[3]{\cos\frac{2n\pi}{3}}$,所以$a_{n}^3=\frac{1}{\log^3n}\cos\frac{2n\pi}{3}$.用狄利克雷判别法级数$\sum_{n=1}^{\infty}a_{n}^3$收敛.(因为$\frac{1}{\log^3n}$递减趋于零, $\cos\frac{2n\pi}{3}$部分和有界).而

\begin{align*}\sum_{n=1}^{\infty}\frac{a_{n}}{n}&=\sum_{k=0}^{\infty}\left[\frac{1}{(3k+3)\log(3k+3)}-\frac{1}{\sqrt[3]{2}}\left(\frac{1}{(3k+1)\log(3k+1)}+\frac{1}{(3k+2)\log(3k+2)}\right)\right]\\&\sim\sum_{k=0}^{\infty}\frac{1}{k\log k}\end{align*}

发散。(柯西积分判别法)

 

如果是正项级数,根据Holder不等式能够说明$\sum_{n=1}^{\infty}\frac{a_{n}}{n}$收敛.